[ 3 / biz / cgl / ck / diy / fa / ic / jp / lit / sci / vr / vt ] [ index / top / reports ] [ become a patron ] [ status ]
2023-11: Warosu is now out of extended maintenance.

/sci/ - Science & Math


View post   

File: 74 KB, 600x838, __miyako_yoshika_touhou_drawn_by_ini_inunabe00__6f7e6e136c70576b1747618ef15413f4.jpg [View same] [iqdb] [saucenao] [google]
11314509 No.11314509 [Reply] [Original]

Formerly >>11300734

>> No.11314603
File: 1.91 MB, 1260x1980, __izayoi_sakuya_touhou_drawn_by_masanaga_tsukasa__3d22998d5b2ab12eead0e0525f2860ba.jpg [View same] [iqdb] [saucenao] [google]
11314603

Unanswered questions:
Math questions:
>>11304190
>>11306549
>>11307710 [Answered in /mg/.]

Physics questions:
>>11302233 [Quite possibly not physics.]
>>11305244

Chemistry questions:
>>11306061

Biology and Medicine questions:
>>11302591

Stupid questions:
>>11304352
>>11304476
>>11304687
>>11305325 [This one's actually me. I think someone is botting up the entire website.]
>>11309573 [Sorta answered, but doesn't count.]
>>11311785
>>11311803
>>11312136 [Answered in /mg/.]
>>11312155 [Autism.]
>>11312212
>>11312231
>>11313776

Advice requests:
>>11311056

Technically answered but feel free to give further insight:
>>11302035
>>11311825

/wsr/ questions:
>>>/wsr/759974 [Answer this one there.]

I feel like I keep changing the formatting for this without noticing.

>> No.11314671

how do people come up with the differential equations to model a phenomenon? Obviously they don't just pull it out of their ass, what is the thought process from going from, say, a scatter plot of data to an equation that models that data?

>> No.11314693

>>11314671
Usually a blend of intuition and empirical data.
>if there are twice as many people, they'll have twice as many children, so [math]p'=kp[/math], where p is the population and k is some value

>> No.11314695
File: 367 KB, 304x209, LOLIDUNNO.gif [View same] [iqdb] [saucenao] [google]
11314695

How do i shot web?

>> No.11314758
File: 31 KB, 658x720, 1526839071904.jpg [View same] [iqdb] [saucenao] [google]
11314758

Scientifically speaking, is there a way to reset my brain so it assumes normal gender and sexuality preferences (i.e. cis and straight?)

>> No.11314764

>>11314758
No. Get used to it~

>> No.11314783
File: 715 KB, 750x1005, expand.png [View same] [iqdb] [saucenao] [google]
11314783

>>11314603
Those math/phys questions should go under the stupid questions category

>> No.11314787
File: 422 KB, 1010x1500, MV5BMzcwYWFkYzktZjAzNC00OGY1LWI4YTgtNzc5MzVjMDVmNjY0XkEyXkFqcGdeQXVyMTQxNzMzNDI@._V1_.jpg [View same] [iqdb] [saucenao] [google]
11314787

>>11314758
Like this movie teaches, hygene of the thought is the key to sanity.

>> No.11315193

Where can I expect to work after studying one of the different fields?
For example if I study math I can probably expect university work, etc.
Which one will put me in the fabled small cabin on the north pole?

>> No.11315199

>>11314787
I always forget to watch this movie

>> No.11315212
File: 128 KB, 1251x381, Stupid question.png [View same] [iqdb] [saucenao] [google]
11315212

How to quicly take the square root of a complex number?
Everytime the textbook skips the steps like it's a natural thing I feel like a dumbass for taking so long.

>> No.11315234

>>11314758
>gender
This will probably get /lgbt/ frothing at the mouth.
But I always thought that if I experienced some form of body disphoria, I would try practicing sports to build a healthier relationship with my body.
In particular, violent sports. Boxing would probably work best. Maybe also something like Rugby.
>sexual preferences
That one's harder. Did you try prayer?
>>11314783
The difference of functors question is weird, but it isn't really stupid.
>>11315212
>Everytime the textbook skips the steps like it's a natural thing I feel like a dumbass for taking so long.
The textbook isn't skipping the steps because they're quick, but because they're really, really boring.
Just pass it into polar form, apply de Moivre's, pass it out of polar form;.

>> No.11315703

according to this video https://www.youtube.com/watch?v=8H0l1u_xQ5o
[math]$\sqrt[3]{1819}$[/math] is 19 but in my text book it results in \simeq12,26
i don't know how to calculate [math]$\sqrt[3]{1819}$[/math]

>> No.11315719

Is it appropriate to use Grug meme in a seminar paper?
It's not really important class, but I don't know.

>> No.11315733

>>11315703
according to this video https://www.youtube.com/watch?v=8H0l1u_xQ5o
[math]/sqrt[3]{1819}[/math] = 19
but my math textbook tells me that [math]/sqrt[3]{1819}[/math] [math]\backsimeq 12,21[/math
i don't know how to calculate [math]/sqrt[3]{something}[/math]

>> No.11315761

>readable version
according to this video https://www.youtube.com/watch?v=8H0l1u_xQ5o
[math]\sqrt[3]{1819} = 19[/math]
but my math textbook shows that [math]\sqrt[3]{1819} = 12,21[/math]

i don't know how to calculate [math]\sqrt[3]{something}[/math]

>> No.11315827
File: 45 KB, 800x314, 800px-20G_centrifuge.jpg [View same] [iqdb] [saucenao] [google]
11315827

are there any commercial g-force training places that people can expose themselves to? I'd love to get blasted to 5Gs but I'm not sure if any places like this exist. No rollercoasters, but similar to pic related

>> No.11315852
File: 154 KB, 615x461, playground-carousel.jpg [View same] [iqdb] [saucenao] [google]
11315852

>>11315827
>are there any commercial g-force training places that people can expose themselves to?

>> No.11315889

>>11315761
>using a comma as a decimal point
the video shows a method for finding the cube root of a number when the cube root is known to be an integer. 1819 does not have an integer cube root so this method cannot be applied.

>> No.11315890

>>11315852
come on it was a serious stupid question and children's carousel isn't what I was looking for. I mean I get that it's dangerous and you'd probably have to force people to sign a liability waiver but I'm still curious.

>> No.11315911

>>11315890
Fighter jets take up civilians all the time. Idk the costs or the barriers to entry, but I've seen jackass members, late night comedians, and other entertainers up there

>> No.11315930

>>11315890
>serious
Who says I'm not serious? I was half-kidding, but you can legitimately subject yourself to huge g's with one of those.
>radius of ~1 meter
>speed of ~60 rpm
>(1 meter) * (1 rev/sec * 2pi)^2 ~ 4 gees
And that isn't even that big, and also not that fast. It isn't hard to imagine how much brain damage you could give yourself.

>> No.11315951
File: 2.71 MB, 1797x2600, __izayoi_sakuya_touhou_drawn_by_hitomin_ksws7544__5db539564e02d4046ac40456e8847523.jpg [View same] [iqdb] [saucenao] [google]
11315951

>>11315719
Hasn't the grug meme been dead for a while?
>>11315827
Did you try building a large, hollow sphere and riding a motorcycle inside it?

>> No.11315986

>>11315951
Kind of, but it's really fitting , I just don't know if it's appropriate. I haven't seen anyone doing it. I don't know what the reaction of professors would be.

>> No.11316034
File: 237 KB, 600x919, __remilia_scarlet_touhou_drawn_by_fkey__e1ff76f556daa54f2d8ab536d04cf0b8.jpg [View same] [iqdb] [saucenao] [google]
11316034

>>11315986
Try to go through your professor's facebook account/webpage to see if he's the kind of autist who would laugh at this sort of stuff.

>> No.11316077

>>11306549
they represent places where the flow generated by your equation (the flow following your phase portrait lines) does not move and stays put. this should make sense to you because they are the points where all the derivatives are 0 (hence, all the speeds of an object following the equation are zero if it begins at that point).
critical points on their own aren't inherently stable. however, we talk about critical points as "stable" or "unstable." a stable critical point is one where, if you move away from it just a little bit, the flow brings you right back toward it. think of the origin in the system x' = -x, y' = -y, or any other sink.
an unstable critical point is one where this may not happen, the flow can take you away. so the hyperbolic flow x' = x, y' = -y has an unstable critical point at the origin because if you move a bit in the x direction the flow pulls you away from the origin (even though it pulls you back if you move a bit in the y direction).
there are different notions of stability, too - what if the flow keeps you close but doesn't converge to the critical point, for example a center portrait (x' = y, y' = -x)? here, when you move a bit away you circle the origin but never get closer to it. hence we have different notions of stability for critical points and talk about things like "asymptotic stability."
how do you find out what the stability of a critical point is? you look at the eigenvalues of the matrix for your system (if linear - if nonlinear you need to linearize your system first).

>> No.11316101

>>11315719
they're going to ask you where you got the funny image from and then you'll have to explain that it came from a pedophile anime-nazi website

>> No.11316103
File: 26 KB, 320x240, Phoneposter.jpg [View same] [iqdb] [saucenao] [google]
11316103

>>11314509
*raises hand*
How do I bulli weebs more effecively?

>> No.11316326
File: 72 KB, 400x322, 400px-Standard_Model_of_Elementary_Particles_+_Gravity.svg.png [View same] [iqdb] [saucenao] [google]
11316326

Do the second and third generation fermions actually contribute anything to the processes necessary for life or are they just kinda there? If they suddenly disappeared without warning would we be in trouble? (assuming this doesn't break all of physics or some shit)

>> No.11316375
File: 291 KB, 640x550, yukari_smile3.png [View same] [iqdb] [saucenao] [google]
11316375

>>11316326
Their half-lives are extremely short due to their large mass, but they nevertheless constitute alternative scattering channels in high-energy processes. In other words, microscopic scattering events consists of these three generational channels for which the first generation would be "less frequent" than if the other generations' particles did not exist. This may have a very small effect in each scattering event, but on a larger scale (such as star formation) it can have significant consequences.
These heavy excitations also contribute to asymptotic freedom but that's more of a theoretical point.

>> No.11316473
File: 106 KB, 1052x1052, 1494998086017.jpg [View same] [iqdb] [saucenao] [google]
11316473

>>11314509
What is needed to identify organic chemical compounds? I've taken two semesters of "general chemistry", and two semesters of "organic chemistry", but i don't have a good answer for this.

In labs we've done stuff like say reagent testing, or column chromatography, and some spectroscopy. We've also read IRs and NMRs in class.

how do you REALLY find out what is in something though? Like, say you've got a cup of gasoline. it's full of mixed aromatic and aliphatic hydrocarbons of different lengths. how do you (or how do people in the industry) determine what is in there, and in what fractions?

My intuitive approach is going to be first separation of compounds (but you'd need perfect separation or it would mess up analysis later) so like a gas chromatograph?
GC-MS is the "gold standard", but why? is that sufficient to analyze like any mix of organic molecules? Why choose it over another one?

I understand how to read the charts, and how to reach the chemical structure, if I know what I'm looking for, but what about new compounds you failed to predict? What if you don't have a reference sample?

>> No.11317174
File: 23 KB, 777x503, Wulong-777x503.jpg [View same] [iqdb] [saucenao] [google]
11317174

NEET wanting to go back to college here. What should i major in undergrad if i want to get into paleontology? I'm assuming it's either geology or biology but i can't just double major.

>> No.11317190
File: 1.12 MB, 3000x2248, IMG_20200120_143306.jpg [View same] [iqdb] [saucenao] [google]
11317190

Could someone please help me with these inequalities?

>> No.11317204

>>11316473
>GC-MS is the "gold standard", but why?
High resolution, high sensitivity. Only really works for compounds you already know
>but what about new compounds you failed to predict?
NMR is the go-to. X-ray diffraction spectroscopy is the king, but it might be hard getting samples/access for it. Either way, you need to purify it first.
>What if you don't have a reference sample?
You need a reference sample.

>> No.11317227

>>11317190
Download and use photomath or something. It's a phone app. If you take a pic of any equation, it'll tell you all the steps and stuff. Pretty convinient.

>> No.11317423

Suppose you cracked public cryptography. How would you sell this knowledge to maximize profit, in either ethical or non-ethical way? Darknet is scary and you are basically breaking the law. But the official reward may not be that much.

>> No.11317750

>>11317423
you would, in all likelihood, be assassinated or disappear

>> No.11317859

does a slope field of a differential equation graph the derivative of the solution, or the solution itself?

>> No.11317880

>>11317859
The slope field is a bunch of little line segments, one for each point on the plane, each of which is tangent to solution of the DE that runs through that point. Basically, it is a visualization of all the possible functions that solve the DE.

>> No.11317891

bls help
You have two equally massive and charged spheres attached to strings; the spheres' charge and mass as well as the length of the strings are known. The spheres can be seen as point particles and their gravity can be neglected.
How do I arrive at the angle at which they stay suspended away from each other? If it changes anything, the angle is known for otherwise equal circumstances but strings double as long.

>> No.11317892

>>11317891
*the gravitational force between them can be neglected

>> No.11317927

>>11317891
These strings are anchored at the same point? Are the charges equal? This sounds like simple statics. Just sum the forces on each mass and set to zero.

>> No.11317930

>>11317891
find the distance at which the electric force and gravitational force between the point charges balances out. Then, use trigonometry knowing that the point where the strings attach forms the tip of two right triangles, with the hypotenuse given and the base*2 solved for.

However, without gravity, they would just spring apart directly opposite each other, assuming like charge.

>> No.11318061

>>11314758
Stop watching anime, it's obviously the issue.

>> No.11318083

is there a easy way of checking whether a graph is connected based only on degrees of vertices?

>> No.11318177

>>11318061
I've never watched anime before. I just have a few pictures saved.

>> No.11318183

If I dug up Einstein's grave and ate him, would I become Einstein?

>> No.11318250

>>11315733
You bullyjerks need to knock it the frick off

>> No.11318348

>>11317859
The whole point of solving a DE is to find the derivative which is the solution. for the 1st order DE that is. For example: y'=f(x,y). so your slope field is a bunch of different f(x,y) depending on the initial conditions. and f(x,y) is the derivative and the solution of this DE.

>> No.11318494

>>11317190
1. The numerator is positive iff 1-√(x+2)>0 => 1>√(x+2) => 1>x+2 => x<-1. The denominator is positive iff 2-√(x+3)>0 => 2>√(x+3) => 4>x+3 => x<1. The inequality is true if the signs differ, i.e. -1<=x<1.

2. log4(x)=(1/2)log2(x), so the inequality can be written
(2-(1/2)log2(x))/(3-log2(x))<=1/2
=> (2-(1/2)log2(x))/(3-log2(x)) <=1/2
=> (4-log2(x))/(3-log2(x)) <=1
=> (u-1)/u <= 1 where u=log2(x)-3
=> u-1 <= u ∧ u>0 or u-1 >= u ∧ u < 0
u-1<=u is always true, u-1>=u is never true, so you have u>0 => log2(x)-3>0 => log2(x)>3 => x>8.

>> No.11318560
File: 285 KB, 941x882, 161039iFB87245FD1D1C064[1].jpg [View same] [iqdb] [saucenao] [google]
11318560

>>11318348
This is a stupid questions thread not a stupid answers thread.
We need to find a _function_ that has a derivative given in the DE. For example y'(x) = x. Following your logic the solution is x. But it is x^2/2+C. It is true that we plot the derivative values on the slope field. But if we connect these points of a given C we will get a _function_ whose derivative we just sketched.

>> No.11318604

I'm taking my first ordinary differential equations class. We just got to separable equations.
For those of you who have taken it,
>what was the most difficult part of the course
>were there any "gotchas" that got you during the semester
>do you have any general tips or advice for someone starting the course

I hear the key to differential equations at a lower level is pattern matching. I haven't seen much that would require it, but reading ahead I see there are different solution paths for DEs of different forms, such as separation of variables, solutions for linear equations using integrating factors, exact differentials, solutions by substitutions...
Is that what they're getting at?

>> No.11318620

Excuse me I'd like a clarification on what is considered baby math please. Would you say linear algebra is at the start of serious math?

>> No.11318634

>>11318560
oh I get it now, because the derivative of a point is the closest linear approximation of the function at that point, so when you plot an infinite amount of points of the derivative, you just get the function itself

>> No.11318637

>>11315761
[math]
\sqrt[n]{x} = x^{ \frac{1}{n}}
[/math]

>> No.11318655

>>11318634
Yes exactly, every point on that plot is a "slope" which is the value of the derivative at that point.
Answering this anon >>11318604
this was probably the most helpful bit to understand. Cause at first I was like dy/dx = F(x,y) wtf is F(x,y)? Turns out you can treat F(x,y) literally as a function of two variables: x,y and the value of that function is your slope at that point! I know it is odd to think about it that way but it works! F(x,y) is essentially a "3d" function: it says give me x and y and I will give you the derivative at that point. You plugin x and y according to your function and the value will be the angle, you can actually use colors if you want. Thats actually how the euler method works, they call it the "euler integration" in the gaming software (but don't use it cause it is inaccurate). And also remember when you sketch the slope field, and your derivative only has one variable, say dy/dx=x, you "fix" and hold a constant value for y, and calculative the derivative for each value of x which is just x in this case: so you keep changing the angle of your slope from left to right: 1 2 3 4 etc. So you don't immediately see your shape, it will be just a bunch of slopes for the given y. Then you change y and repeat the loop for x, and so forth. Once you have a bunch of "rows" for a few values of y, you be like ah I see the pattern. You can connect the slopes according to the pattern and according to the initial condition C.

>lied to captcha about motorcycles again
>i only get motorcycles wtf

>> No.11318659
File: 158 KB, 1047x1117, __flandre_scarlet_and_remilia_scarlet_touhou_drawn_by_gotoh510__bacbb155935feaf2e47184c312d266d3.jpg [View same] [iqdb] [saucenao] [google]
11318659

>>11318620
If I learned it more than a semester ago, it's stuff for children.
If I'm studying it right now, it's proper math for adults.
If I plan on studying it later, it's highly advanced, cutting edge stuff.
>linear algebra
I literally learned linear algebra before figuring out I could use my left hand to count higher than five.
It's womb-tier maths.

>> No.11318674

>>11318659
>I could use my left hand to
hmm, left really
>count higher than five.
ah never mind

>> No.11318741
File: 68 KB, 800x736, 7863a49b4cf2085bf2e84ecc2a64fed5.jpg [View same] [iqdb] [saucenao] [google]
11318741

>>11318604
>what was the most difficult part of the course
Being able to come up with integrating factors by inspection, trying to wrap my head around "variation of parameters."
>were there any "gotchas" that got you during the semester
Maybe not a "gotcha" but like a couple weeks into the course and realizing why lin alg is taught alongside ODEs (common in America). Also, the Wikipedia explains VoP better than any text I was ever prescribed in school.
>do you have any general tips or advice for someone starting the course
In the words of a controls professor quoting his math teacher of some couple decades ago: "Solving a differential equation is glorified guesswork." Don't sweat it it, it's largely a memorization game. I recommend putting more effort into understanding and learning the various numerical methods (of course, you still need to pass this course, anon). Obviously, do lots of practice problems.
>>11314787
Never thought about how cute Russel Crowe is. Is this movie actually good?

>> No.11318923

>>11314671
intuition and then trial and error. thats the "scientific" method. you need nothing solid to make a hypothesis but you need numbers to back them up

>> No.11318927

>>11314758
stop brosing 4chan unironically

>> No.11318933
File: 26 KB, 761x490, gasoline.png [View same] [iqdb] [saucenao] [google]
11318933

hello can you help me with this shit?? I am supposed to solve this using bernoulli. I got the first excersize by using that the velocity of the big area thing is much less than the one in the pipe BUT i cant get the other question. Is there any way of doing the 1st one without that assumption i made??

>> No.11318940

>>11315234
rugby is for rich homos in my country

>> No.11319012 [DELETED] 
File: 246 KB, 791x550, 8ff.png [View same] [iqdb] [saucenao] [google]
11319012

>>11318933
Yes, you are supposed to use Bernoulli. We will take z_1=0 and p_1=101 kPa. From Bernoulli's principle,
[eqn] \frac{p_1}{\rho}+\frac{v_1^2}{2}=\frac{p_c}{\rho}+\frac{v_c^2}{2}+z_cg [/eqn]
Notice that because the container is very large in comparison to the pipe, v_1 is about zero. Also, because both points 1 and c are exposed to the atmosphere, they cancel out. Rearrange to get
[eqn] v_1=\sqrt{-2z_cg}=3.13\text{ m/s} [/eqn]
The next step is important. Realize that because of the implicit assumption that the fluid is incompressible we have it that [math] v_h=v_c [/math]. Also realize that *we cannot have a vacuum* at the apex of the pipe. So [math] p_h>0 [/math]. Apply Bernoulli again:
[eqn] \frac{p_h}{\rho}+\frac{v_h^2}{2}+hg=\frac{p_c}{\rho}+\frac{v_c^2}{2}+z_cg [/eqn]
Cancel the velocity terms and rearrange into an inequality:
[eqn] p_h=p_c+\rho z_cg-\rho hg>0\implies h<\frac{p_c}{\rho g}+z_c=15.1\text{ m} [/eqn]
>Is there any way of doing the 1st one without that assumption i made?
Not unless they give you the dimensions of the tank. And even then, it is very fucky and you will end up with a (non-linear?) differential equation since the pressures and linear head changes with time.

>> No.11319020
File: 452 KB, 661x819, 1571177158892.png [View same] [iqdb] [saucenao] [google]
11319020

>>11318933
Yes, you are supposed to use Bernoulli. We will take z_1=0 and p_1=101 kPa. From Bernoulli's principle,
[eqn] \frac{p_1}{\rho}+\frac{v_1^2}{2}=\frac{p_c}{\rho}+\frac{v_c^2}{2}+z_cg [/eqn]
Notice that because the container is very large in comparison to the pipe, v_1 is about zero. Also, because both points 1 and c are exposed to the atmosphere, they cancel out. Rearrange to get
[eqn] v_1=\sqrt{-2z_cg}=3.13\text{ m/s} [/eqn]
The next step is important. Realize that because of the implicit assumption that the fluid is incompressible we have it that [math] v_h=v_c [/math]. Also realize that we cannot have a vacuum**** at the apex of the pipe. So [math] p_h>0 [/math]. Apply Bernoulli again:
[eqn] \frac{p_h}{\rho}+\frac{v_h^2}{2}+hg=\frac{p_c}{\rho}+\frac{v_c^2}{2}+z_cg [/eqn]
Cancel the velocity terms and rearrange into an inequality:
[eqn] p_h=p_c+\rho z_cg-\rho hg>0\implies h<\frac{p_c}{\rho g}+z_c=15.1\text{ m} [/eqn]
>Is there any way of doing the 1st one without that assumption i made?
Not unless they give you the dimensions of the tank. And even then, it is very fucky and you will end up with a (non-linear?) differential equation since the pressures and linear head changes with time.

****The situation is actually worse than this. We cannot have a pressure lower than the vapor pressure of the fluid at that point. When you do, you get these little exploding gas bubbles called CAVITATION which will end up seriously fucking up your pipes and any pumps/turbines/equipment you have.

>> No.11319030

>>11319020
thank you bronie

>> No.11319033

If I have a set {0,1} and and define addition and multiplication as in boolean algebra (i.e., 0 * 1 = 0, 1 + 1 = 0, etc), is there any way to show that this is a field other than brute forcing my way through every combination to show it satisfies the field axioms?

>> No.11319100

>>11318604
>difficult
handling lots of messy derivations and coefficients for inhomogeneities (variation of parameters / duhamel, ansatz with undetermined coefficients)
>gotchas
probably remembering what to do when you cant diagonalize the matrix for systems
>tips
keep drilling all the strategies so that you aren't looking back at something for the first time in months come the final. read every word / modeling problem carefully. get comfortable with creating ansatz solutions and recognizing when an integrating factor could work. review your linear algebra, specifically diagonalization, if you're going to be doing systems of ODEs.

>> No.11319101

>>11319033
But that's not how + is defined in boolean algebra. In boolean algebra, + is OR, so 1 + 1 = 1, and you don't have a field.
If you mean the integers mod 2, then sure, but showing the field axioms is incredibly easy.

>> No.11319176
File: 266 KB, 428x556, yukari_smile1.png [View same] [iqdb] [saucenao] [google]
11319176

>>11318083
If [math]G[/math] has no self-loops, let [math]n_\pm(v)[/math] be the number of incoming (outgoing) edges at the vertex [math]v\in V[/math], put [math]b_0(G) = \frac{1}{2}\sum_{v\in V}\left(n_+(v) - n_-(v)\right)[/math].
Consider [math]G[/math] as an oriented 1-skeleton of a 2D CW complex [math]X[/math]. The invariant [math]b_0(G)[/math] is the same as the invariant[math]\operatorname{rank}H_0(X,\mathbb{Z})[/math] of the cellular homology of [math]X[/math].

>> No.11319215

Apparently we know that neutrinos aren't massless because they can change flavors, which means they experience the passage of time and thus don't travel at the speed of light. But can't high energy photons randomly transform into particle anti-particle pairs? Doesn't that mean they experience the passage of time as well? What makes these two transformations different?

>> No.11319271
File: 1.03 MB, 1659x2304, yukari_pills.jpg [View same] [iqdb] [saucenao] [google]
11319271

>>11319215
First of all, photons are bosons with spin-0 and classically obey the Klein-Gordon equation [math]\square A = 0[/math], while quarks are fermions with spin-1/2 obeying Dirac equation [math](\not\partial -im)\psi = 0[/math]. There is no concept of "flavour" for photons, or any other gauge fields for that matter.
Second of all, massless bosons exhibit full internal gauge symmetry while massless fermions only exhibit a [math]U(1)[/math] axial symmetry. Mass terms gap out the spectrum and leads to cross terms [math]m^2\psi_f^\dagger \gamma^5 \psi_{f'}[/math] that contribute to scattering between quarks of different flavours. No such thing occurs for photons; pair creation does not require a spectrum-gapping mass term to occur,. It's like the first vertex [math]\psi^\dagger \not A \psi[/math] you see in QED.

>> No.11319354
File: 57 KB, 500x353, 1482851479278.jpg [View same] [iqdb] [saucenao] [google]
11319354

What is it that makes it true in the integers that if a < b, then a + 1 <= b? I want to use this fact in a proof for the divsion algorithm. Is well-ordering enough? Or does it follow from the construction of the integers themselves?

>> No.11319387

>>11319354
Nevermind found the answer.

>> No.11319497

is there a program I can use to visualise 3d vectors? I specifically want to input parametric equations like (a,b,c)+t(d,e,f)

>> No.11319555

How do I git gud at integration by substitution?
Even basic stuff like sqrt(r2-x2) integrated over x (/sci/let, sorry) seems completely unintuitive to me looking at the actual antiderivative.

>> No.11319744
File: 17 KB, 921x182, 957689b534c9e8442068f7bf001e65d9.png [View same] [iqdb] [saucenao] [google]
11319744

How do I prove this by induction? I know the base case (n = 2), which is true. But when it comes to the inductive step I get stuck, how do i get it to n+1?

>> No.11319788

>>11319744
when doing the inductive step you assume that the statement is true for P(n) and show that it must also be true for P(n + 1). In most cases you want to take the LHS of the n+1 statement and turn it into "something + LHS of n" or similar. In your problem this is really easy to do and it basically solves itself once you get it into this form.

>> No.11319809
File: 94 KB, 850x531, __patchouli_knowledge_and_remilia_scarlet_touhou_drawn_by_terimayo__sample-ad96284c916a2e5c6b26afd100f16342.jpg [View same] [iqdb] [saucenao] [google]
11319809

>>11318083
For a graph G with n vertices, if a vertex a has degree k, then the connected component it's in needs to have at least k+1 vertices. For another vertex b, if it has degree larger than n-k-1, then it's connected component needs to have more than n-k vertices.
Assuming a and b are on distinct connected components, you have n>k+1+n-k=n+1, and thus a and b are in the same component, by contradiction. If all vertices other than a have degree larger than n-k-1, then all vertices are on the same connected component as a, and the graph is connected.
>>11318674
Lewd.
>>11319744
Set [math]\Sigma_{k=1} ^n x_k = a[/math]. Then the new LHS for the induction becomes [math](a+x_{k+1})^2 = a^2 + 2ax_{k+1} + x_{k+1} ^2[/math].

>> No.11319815

>>11319744
[math]\sum_{k=1}^{n+1}x_k=x_{n+1}+\sum_{k=1}^n x_k[/math]
Likewise for the other sums. So when incrementing the upper limit of a sum, you replace it with the original sum plus the new element. When you distribute multiplication over addition, you'll end up with the original expression being one of the terms.

>> No.11320003

What time would it take to empty a 1 meter long 5cm diameter pipe fulll of water take to empty if opened?
I wuld use directly the same formula i would use for any solid body but using bernoulli seems to go to a dead end...

Have any books on waves (physical waves, physical optics, cords, etc) that you recomend?

>> No.11320322
File: 134 KB, 600x635, 8b773db741ce8b1140022a3a06545403.jpg [View same] [iqdb] [saucenao] [google]
11320322

>>11320003
>bernoulli
No, you can't use Bernoulli in the case of non-steady flow. For a problem like this you could take the computational fluid dynamic (CFD) approach, which is hard, or you could use something based and redpilled called the Buckingham Pi theorem.
First, think about what variables the time to empty the pipe would depend on. I can come up with six: time [math] t [/math] depends on the length of the pipe [math] \ell [/math], area of the pipe [math] A [/math], hydraulic diameter (to account for more x-sections than a circular pipe) [math] D [/math], gravity [math] g [/math], viscosity [math] \mu [/math], and density of fluid [math] \rho [/math]. So we can say there is some function [math] f(t,\ell,A,D,g,\mu,\rho)=0 [/math]. Our goal is to non-dimensionalize this function. We know that [math] f [/math] depends on 3 fundamental dimensions: length, mass, and time. The Buckingham Pi theorem says we can therefore develop a new function of 7-3=4 dimensionless terms called pi terms.
[eqn] F(\pi_1,...,\pi_4)=0\implies\pi_1=G(\pi_2,...,\pi_4) [/eqn]
When you go through the algebra you get
[eqn] \frac{t\mu}{\rho\ell^2}=G\Bigg(\frac{A}{\ell^2},\frac{D}{\ell},\frac{g\ell^3\rho^2}{\mu^2}\Bigg) [/eqn]
for which at this point you would do some experimentation to find exactly what this function[math] G [/math] is. Once you get that, you can find time for any shape and length of pipe for whatever fluid you like.

>> No.11320415
File: 62 KB, 415x573, thot patrol monk.jpg [View same] [iqdb] [saucenao] [google]
11320415

Can I be a materialist atheist who believes on my own immortality trough quantum immortality,or robert lanza's biocentrism?

>> No.11320427

>>11320415
No. Quantum immortality is just new age bullshit with no basis on reality or science. It's no better than just believing in god.

>> No.11320434

>>11320427
i REJECT your no

>> No.11320443
File: 89 KB, 1072x739, Untitled.png [View same] [iqdb] [saucenao] [google]
11320443

can someone explain why what I did in red is wrong? I dont see why the derivative of the function needs to be taken at all when only the lengths/distances are needed to find the angle

>> No.11320449 [DELETED] 

>>11320443
Why are you allowed to simply use the reciprocal of the derivative like that?

>> No.11320462

>>11320443
fuck nevermind

>> No.11320464
File: 72 KB, 800x721, a6e54dba22aa751fce4897ca4996be61.jpg [View same] [iqdb] [saucenao] [google]
11320464

>>11320443
Because the strain is obviously different at different points along the block. dx/dy=γ, approximately.
>>11320449
Because derivatives are just the ratio of differential quantities :^]

>> No.11320469
File: 287 KB, 1540x1482, _junko_touhou_drawn_by_kaiteki_gk428425_1502f8afd47bb37dcbbe7dddd6b8c962.png [View same] [iqdb] [saucenao] [google]
11320469

>>11320449
https://en.wikipedia.org/wiki/Inverse_function_theorem

>> No.11320479

>>11320434
If you're going to believe in silly modern religion for dumb fedoras i can't really stop you.

>> No.11320517

>>11320479
i will DISCOVER a method to quantum dimension jump
>>11320469
are this girls from that bullet hell game cirno is from?

>> No.11320542
File: 22 KB, 224x224, 1008.png [View same] [iqdb] [saucenao] [google]
11320542

Physics, Math, or Chemistry?

>> No.11320589
File: 59 KB, 640x640, 9B47734B78DE455FA9A6EA6377719331.jpg [View same] [iqdb] [saucenao] [google]
11320589

>>11314509
Hey, I never posed in /sci/ ever, but I was just researching math for some reason.

If I get this right, math is actually super easy, its just that the way we teach it to kids is fucking everything up?
>is this corret? and if not, what is the corret way to describe what i just tried to say?

And what reason is there to lean it if one is not in school? (if i dont want to do a job that's reliant on it)
>Is there a philosophical reason to be good at it?

I want to be good at it for some reason, maybe I feel left out or its making me feel stupid if i don't understand what everyone here is posting.

and finally,
>are humans who are good at math, smarter humans?

>> No.11320591

>>11314509
What is the pattern here? Is there a formula for the ith element in the jth row?
[
[0, 24, 50, 78, 104, 132, 158, 182],
[0, 23, 45, 68, 90, 113, 135, 158],
[0, 19, 38, 57, 75, 94, 113, 132],
[0, 14, 29, 45, 59, 75, 90, 104],
[0, 10, 21, 33, 45, 57, 68, 78],
[0, 5, 12, 21, 29, 38, 45, 50],
[0, 1, 5, 10, 14, 19, 23, 24],
[0, 0, 0, 0, 0, 0, 0, 0]
]

>> No.11320784

>>11315234
>I always thought that if I experienced some form of body disphoria, I would try practicing sports to build a healthier relationship with my body.

Hypermasculinity is a common coping strategy. I know three mtf women with combat experience, two have purple hearts, one is a retired seal.

>> No.11320787

>>11320469
I figured it out 15 seconds after asking but thanks anyway

>> No.11320792

>>11315827
Could you find a not-busy theme park with a Gravitron?

>> No.11320797

>>11317423
I wouldn't sell it. I'd sell Satoshi's bitcoin's instead.

>> No.11320843

Let [math]X[/math] be an almost-surely bounded random variable. Does its expectation necessarily exist?

I get that *if* it does, then it must be finite (and no larger than the bound over [math]|X|[/math]) - but must the expectation converge? Can't it diverge by oscillation somehow? (If so, I'd love to see a concrete example)

>> No.11320865

>>11320843
>Can't it diverge by oscillation somehow?
It shouldn't, because it's measurable.
Anyhow, can you state the problem in terms of measure theory?

>> No.11320899

>>11320865
Ah. I see. I don't know much about measure theory - I'm taking an introductory probability theory class, which bypasses most of the MT-related heavy lifting. I do my best to study up on MT online by myself, but I'm only familiar with the most basic definitions of the field. So if I understand you correctly, any *bounded* measurable function is integrable? (I know there exist expectation-less rvs, which are (?) measurable functions, so I suppose the boundedness is critical?)

>> No.11320918

Am I understanding radioactive decay correctly?
>Each particle or system has a "most probable" energy value
>The farther the system is from this value, the "rarer" it becomes
>Every system has a chance to decay into other systems with the same charge/spin/energy, and this chance is determined by the difference in "rarity" between the two
I feel like I'm definitely missing something, because wouldn't this mean that having knowing the charge, spin, and energy of any given isolated system, would mean knowing as much as you possibly could know about that system?

>> No.11320934

>>11320899
>any *bounded* measurable function is integrable?
No, the constant functions give an immediate counterexample.
But if it's something like "a bounded measurable function times a probability density", I think that worked.

>> No.11320937

According to you guys, when is one considered too old to attend university? Can one have too many degrees?

>> No.11320942

>>11320934
Here we go:
https://math.stackexchange.com/questions/507335/product-of-measurable-and-integrable-functions
I'm not sure if this is what you want, tho, but I'm really not in the mood to carefully nitpick my way through probability theory's definitions.

>> No.11321003
File: 7 KB, 312x175, 1579650386.png [View same] [iqdb] [saucenao] [google]
11321003

"perfect low pass don't exist"
"design a good filter that reduces the frequencies you don't want well, and keeps the one you want to keep"
Why can't we just we just do a fourier transform to get the frequencies of a signal,
then remove the frequencies we don't want by setting them to zero,
and then turn it back into a signal with an inverse fourier transform.

Or, simpler asked, why can't I just tell an audio program to "remove all frequencies under 500 Hz and don't touch the rest"

>> No.11321033 [DELETED] 
File: 349 KB, 1000x809, __remilia_scarlet_touhou_drawn_by_chobi_sakuyasakuhana__3d4377480d5fef9892d451481d497af2.jpg [View same] [iqdb] [saucenao] [google]
11321033

>>11321003
Wild guess says it's because the signal isn't exact.

>> No.11321039 [DELETED] 

>>11321033
Wild guess was wrong.
But wikipedia actually discusses it.
https://en.wikipedia.org/wiki/Low-pass_filter#Ideal_and_real_filters

>> No.11321044 [DELETED] 

>>11321003
>remove the frequencies we don't want by setting them to zero
"Removing frequencies we don't want" is exactly what a filter does, love. Filters are how we accomplish this.

>> No.11321059

>>11321003
"Removing frequencies we don't want" is exactly what a filter does, love. Filters are how we accomplish this. Perfect filters don't exist for the same reason a function generator can't generate perfect signal forms.

>> No.11321066

>>11321003
The answer is simpler than you think, you cannot apply the convolution you described perfectly without an infinite time domain. It is perfectly possible if you extend your domain somehow, for example just repeat your discrete signal infinitely in both directions.

>> No.11321080
File: 16 KB, 567x498, remfirls.gif [View same] [iqdb] [saucenao] [google]
11321080

>>11321059
>"Removing frequencies we don't want" is exactly what a filter does
Yes, but it also touches the rest.
For example, in the picture, all those waves on the left. It's supposed to be a constant 1.0 there, but all over the place.
Why can't we just be like
>if f < 500Hz: 1
>if f >= 500Hz: 0
>>11321066
>You cannot apply the convolution you described perfectly without an infinite time domain.
Could you elaborate on this part
Is it because in order to get 0.1 Hz you need at least 10 seconds of sample time, so to cover all frequencies, you need infinite time?

>> No.11321088

>>11320918
Don't think about it as some abstract "rarity", it's only about entropy, i.e. the distribution of energy.

>wouldn't this mean that having knowing the charge, spin, and energy of any given isolated system, would mean knowing as much as you possibly could know about that system?

if you are only concerned with final state (heat death) then yeah, but usually the intermediate behavior is plenty interesting

>> No.11321090

>>11321080
>Why can't we just be like
>>if f < 500Hz: 1
>>if f >= 500Hz: 0
Please explain how you could have hardware that does this without a filter at some level.

>> No.11321104

>>11321090
>input signal
>ADC
>FFT
> if / else
>IFFT
>DAC
>output signal

I'm assuming that if all the parts of this chain had a very high resolution, it would be near perfect. But I have no idea, that's why I'm asking.

>> No.11321108

>>11321080
>Is it because in order to get 0.1 Hz you need at least 10 seconds of sample time, so to cover all frequencies, you need infinite time?
Yeah that's a good way to think of it intuitively. To justify it formally you just have to observe that the fourier transform of the perfect square wave (sinc) has value in an infinite domain.

>> No.11321124

>>11321003
I was wondering about the same thing. Basically "frequency domain filter". Imagine you have a smoothie and you want to remove ingredients you don't want. You get the components, remove some, and then combined them together again and maybe add your own.
>>11321059
> is exactly what a filter does
So all filters do a fourier transform?

>> No.11321134
File: 17 KB, 560x420, ukOfZ[1].png [View same] [iqdb] [saucenao] [google]
11321134

What is the relationship between the frequency response, spectrum, and transfer function? All mean the same thing? This type of a Bode plot?
Is it as if we fed a square pulse into our filter?

>> No.11321137

>>11321134
> square pulse
i mean the dirac delta (impulse) function?

>> No.11321146
File: 6 KB, 474x178, 1579655245.png [View same] [iqdb] [saucenao] [google]
11321146

>>11321124
>You get the components, remove some, and then combined them together again
You put it better than I did. That's exactly what I was thinking.
>>11321108
>sinc
That makes sense to me. I have it in my lecture notes too.
But why does the filter in this picture look so bad: >>11321080
As far as I know, filters get less wavy at an higher order, so I assume if the order was really high, it would almost be rectangle shape?

And another thing, the screenshot in my post are the low pass filter options that I can select in audacity for audio editing.
Are those 5 options just different orders of filters? Why can't I just use something like order 1,000,000 and computer for a few minutes to get the perfect result.

>> No.11321150

hehe i didn't realize which thread i was in, i thought this was /diy/ /ohm/, good discussion.

>> No.11321152

>>11321104
>ADC
So a filter

>> No.11321153

Why do they say that the Fourier transform is a function of a _real_ variable when it is actually complex, just one dimensional, jw. Can think of it as 0+jw right? Which is a PI/2 shift. Why this phase shift? How is it real. So confusing.

>> No.11321159
File: 172 KB, 1137x1368, __remilia_scarlet_touhou_drawn_by_rin_falcon__bda8e21f753a1930dd3f55827b44987b.jpg [View same] [iqdb] [saucenao] [google]
11321159

>>11321150
>look up ohm
This does remind me:
https://www.strawpoll.me/19285141

>> No.11321160

>>11321152
As far as I know, an ADC uses comparators to quantize the signal, which is kinda like if/else.
Can you tell me where there's a filter in an adc because I don't know

>> No.11321169

>>11321159
i answered yes but i know nobody reads it. cognitive dissonance right there. hmm

>> No.11321175

>>11321169
We all like to pretend sometimes.

>> No.11321186
File: 98 KB, 814x960, 1492579443676.jpg [View same] [iqdb] [saucenao] [google]
11321186

How do I decline a previously (although informally) accepted PhD offer? I have found some helpful threads and articles on the internet about this, but my case has an extra factor. The potential adviser (whose offer I am declining) is a main collaborator in a current project and collaborates with my current advisor (I'm in a Masters program) in many other projects.

Any advice on how to write a good letter?

>> No.11321193

>>11321003
> "perfect low pass don't exist"
Perfect low-pass is non-causal, i.e. the current output depends upon *future* values of the input.

> Why can't we just we just do a fourier transform to get the frequencies of a signal, then remove the frequencies we don't want by setting them to zero, and then turn it back into a signal with an inverse fourier transform.
Latency. To implement a non-causal filter, you have to delay the output w.r.t. the input so that you have future inputs available.

A perfect low-pass filter is a Heaviside step function in the frequency domain, which corresponds to convolution with a sinc function in the time domain. But a sinc function has infinite extent, so a perfect low-pass filter would require values infinitely far into the future, meaning that you'd need infinite delay.

Truncating the filter to get a finite delay (which is exactly what Lanczos does) means that you no longer have a perfect low-pass filter.

The approach you suggest works fine if you're filtering e.g. a WAV file, where you can use the last input sample in the calculation of the first output sample, but can't be used in real-time where you have an upper bound on the permissible delay.

>> No.11321203

>>11321193
But this is so strange. For example the walls and various obstacles can act as realtime sound filters by blocking certain frequencies, They don't know or care they are non-casual.

>> No.11321209

>>11321193
Thanks, I get it now, that was the answer I needed.
The connection between causality, buffer and files that are already written explained it to me, because I was wrongly assuming a filter that works files works the same in real-time.
I have no more questions, but I also find this interesting while we're at it: >>11321203

>> No.11321213

>>11321160
Sampling is equivalent to modulation with a repeating dirac delta function followed by a box filter.

It's not strictly a "filter" in the sense of a linear time-invariant (LTI) transform. But any practical DSP system is also going to have a low-pass filter on the input to prevent aliasing (any input components above half the sampling frequency will produce low-frequency components due to modulation) and another on the output to eliminate quantization noise.

>> No.11321219

>>11321209
I guess I asked this question >>11321203 but then realized we are talking about "perfect" filters but while the walls do block sound I don't know their "transfer function" maybe they are horrible filters.

>> No.11321221

>>11321203
They aren't perfect low-pass filters. Higher frequencies are attenuated at some dB/octave slope, not completely eliminated above some cut-off frequency.

>> No.11321240
File: 94 KB, 300x450, 1493310600988.png [View same] [iqdb] [saucenao] [google]
11321240

>>11321186
Apologize and explain to them briefly what made you change your mind. Assuming you applied for a position in the upcoming Fall semester that should suffice.

>> No.11321283

>>11321240
It is indeed for the Fall semester. How much apologizing you reckon is in order and what if the reason is somewhat involved... How much detail should I give?

>> No.11321297
File: 718 KB, 1200x1600, __remilia_scarlet_touhou_drawn_by_gla__f60b4cea38be8d4e53e6b223e045a7ed (1).png [View same] [iqdb] [saucenao] [google]
11321297

Any and all feedback will be appreciated.
Specially suggestions/links to convenient resources.
I've already asked /g/'s /sqt/ if they have any better websites for files. Hopefully they deliver.

>what is /sqt/ for
Questions relating to math and science, plus appropriate advice requests.
>where do I go for other SFW questions and requests?
>>>/wsr/ , >>>/g/sqt , >>>/diy/sqt , >>>/adv/ , etc.
>pdfs?
libgen.is (Warn me if the link breaks.)
>book recs?
https://sites.google.com/site/scienceandmathguide/
https://4chan-science.fandom.com/wiki//sci/_Wiki
>how do I post math symbols?
https://imgur.com/a/mIhqgkm

Question asking tips and tricks:
>attach an image
>if you've made a mistake that doesn't actually affect the question, don't reply to yourself correcting it. Anons looking for people to help usually assume that questions with replies have already been answered, more so if it has two or three replies
>ask anonymously
>check the Latex with the Tex button on the posting box
>if someone replies to your question with a shitpost, ignore it

Resources:
Good charts: https://mega.nz/#F!40U0zAja!cmRxsIoiLFZ_Mvu2QCWaZg
Shitty charts: https://mega.nz/#F!NoEHnIyT!rE8nWyhqGGO7cSOdad6fRQ
Verbitsky: https://mega.nz/#F!80cWBKxC!ml8ll_vD2Gbw4I1hSLylCw

>> No.11321301
File: 405 KB, 1474x1328, __yakumo_yukari_touhou_drawn_by_nameo_judgemasterkou__d361e7aa66a282581994c23b9906b59b.jpg [View same] [iqdb] [saucenao] [google]
11321301

>>11321283
Typically for me, if I was unable to make a conference or meeting (which is presumably less sever than your situation), I apologize right at the beginning and at the very end.
>How much detail should I give?
Beyond briefly describing the issue, I'd only provide details about things that are immediately relevant to the professor.

>> No.11321345

>>11321297
Forgot to add desmos and wolfram to resources.
Will put it in later.

>> No.11321359

>>11321297
add google scholar and the /sci/ archive with a suggestion to use either if the question can be answered more clearly by a research paper/book or has been answered. the repeat questions are a waste of time

>> No.11321514

If v is perpendicular to w, find a vector u such that

u cross v = w.

Any ideas? I was thinking u = (v cross w)/v^2 but I have no clue how to verify it.

>> No.11321586

how do you solve trig equations to find where they intersect without a graph?

(secx)^2 = 8cosx

>> No.11321691

what's the difference between grad school and masters?

>> No.11321711

>>11314509
What would you recommend for someone who needs to study math and logic and hasn't done so in a great while?

>> No.11321720

>>11321711
>logic
just think an extra 5 seconds before you take your own words or the words of others seriously
>math
read the sticky carefully, follow the link to the wiki page and make use of the ample resources available to regain basic mathematical competence. Should take less than 6 months of moderate focused effort

>> No.11321737

>>11321586
It depends entirely upon the equation. This one is trivial, as sec(x)=1/cos(x), so
sec(x)^2=8*cos(x)
=> 1/cos(x)^2=8*cos(x)
=> cos(x)^3=1/8
=> cos(x)=1/2
=> x=π/3+2nπ or x=5π/3+2nπ

>> No.11321756

It’s been 8 days since my interview and I haven’t heard back.
They said I would hear from them. Should I still send a follow-up email?

>> No.11321863
File: 761 KB, 320x240, 1573510817079.gif [View same] [iqdb] [saucenao] [google]
11321863

Has anybody tried Adderall while being depressed? I don't want to make it too long but basically I've been struggling with depression for so many time and going to college having depression is almost impossible, I can barely leave my bed, when I do I can barely leave my home (I also have a really bad anxiety issue), I almost never go to any kind of classes, sitting to study is almost impossible and when I do I find really hard to do it for more than half an hour cause my self harming thoughs are unbearable, and I basically stopped going to uni cause I felt so ashamed of going for two weeks and then stopping everything. Now I'm trying to study for an exam but as I said I find so hard to concentrate, and when I try to concentrate I usually end up crying cause my stupid mind starts thinking in horrible things. I never took Adderall, I always thought it will be bad for my head that it's kinda like cheating. But at this point of my life I would accept everything. I know I don't have ADHD but maybe it could also help depressed people to concentrate. I would really appreciate if someone could share it experience.

>> No.11321871

>>11321863
I forgot to add I don't want to take them regularly, but maybe 2 weeks before a final and around 4/5 days before a regular exam help me to improve my performance and make me feel less frustrated.
Besides, I don't have a really addictive personality.

>> No.11322012
File: 98 KB, 667x431, mt.png [View same] [iqdb] [saucenao] [google]
11322012

Is it possible to learn algebra 1/2, trig and these subjects by July? What about next year?

>> No.11322036

Say I have two positive bounded sequences, not necessarily convergent, what operations can I do with them so that the resulting sequence is also bounded?

>> No.11322047

>>11322012
Read Lang's Basic Mathematics + A First Course in Calculus and see how quickly you go.

>> No.11322116

Could a light gas gun (with no barrel or projectile, just the piston, gas and rupture disk) be used as a primer for a secondary type high explosive? Or is a very fast burst of inert gas like that not enough?

>> No.11322183

>>11322012
Yes I think so...
I'm not sure about algebra and trig, but you should be able to produce a basic apprehension of calc and linear algebra in a short while. Calculus should take about 8-12 hours and linear algebra 8-12 hours. Then its just a task of remembering the material by reviewing course material as need be and doing practice problems.

>> No.11322297

How to show if (a,b)|c => (a,b)|(b,c)?

>> No.11322341

>>11321514
https://en.wikipedia.org/wiki/Triple_product#Vector_triple_product
a×(bxc) = (a·c)b-(a·b)c
If a=b=v, c=w
=> v×(v×w) = (v·w)v-(v·v)w
If v is perpendicular to w then v·w=0 and the above reduces to
v×(v×w) = -(v·v)w = (-||v||^2)w
The cross product is anti-commutative: a×b=-b×a, so
(v×w)×v = -v×(v×w) = (||v||^2)w
=> ((1/(||v||^2))(v×w))×v = w

>> No.11322416
File: 6 KB, 568x115, af867709396ec02f398d8c70615500e7.png [View same] [iqdb] [saucenao] [google]
11322416

Given this, how do I prove that if x_n is in (0, sqrt2), so is x_(n+1)? It seems simple but I feel like I'm missing something

>> No.11322512

>>11322047
Lang’s Calculus book sucks

>> No.11322531

>>11322416
If x_n=0 then x_n+1=1/2. If x_n=sqrt2 then x_n+1=sqrt2. Notice that f(x_n)=x_n+1 increases monotonically on (0, sqrt2) and that nowhere on this interval is its derivative zero. Also realize that extrema for a function on an interval can only be found at the boundaries or where the derivative is zero. Since the boundaries are in [0, sqrt2], we have proved the range of function is within the domain.

>> No.11322596

If i did electrical engineering in undergrad can i still go to grad school for physics?

>> No.11322605

If you had a black canvas with red paint on it, or a white canvas with the same red paint on it, the red on the white canvas would stand out more. Why is this? Wouldn't the black with no light reflecting make the red stand out more?

>> No.11322653

>>11321359
Better?

>what is /sqt/ for
Questions relating to math and science, plus appropriate advice requests.
>where do I go for other SFW questions and requests?
>>>/wsr/ , >>>/g/sqt , >>>/diy/sqt , >>>/adv/ , etc.
>pdfs?
libgen.is (Warn me if the link breaks.)
>book recs?
https://sites.google.com/site/scienceandmathguide/
https://4chan-science.fandom.com/wiki//sci/_Wiki
>how do I post math symbols?
https://imgur.com/a/mIhqgkm
>a google search didn't return anything, is there anything else I should try before asking the question here?
https://scholar.google.com.br/
>where do I look up if the question has already been asked here?
>>/sci/
https://boards.fireden.net/sci/


Question asking tips and tricks:
>attach an image
>if you've made a mistake that doesn't actually affect the question, don't reply to yourself correcting it. Anons looking for people to help usually assume that questions with replies have already been answered, more so if it has two or three replies
>ask anonymously
>check the Latex with the Tex button on the posting box
>if someone replies to your question with a shitpost, ignore it

Resources:
Good charts: https://mega.nz/#F!40U0zAja!cmRxsIoiLFZ_Mvu2QCWaZg
Shitty charts: https://mega.nz/#F!NoEHnIyT!rE8nWyhqGGO7cSOdad6fRQ
Verbitsky: https://mega.nz/#F!80cWBKxC!ml8ll_vD2Gbw4I1hSLylCw
Graphing: https://www.desmos.com/
Calc problems: https://www.wolframalpha.com/

>> No.11322701

>>11314758
>Scientifically speaking
No
>Is there a way to reset my brain so it assumes normal gender and sexuality preferences
Yes

>> No.11322736

>>11322653
>https://imgur.com/a/mIhqgkm
Is this a thumbnail?

>> No.11322741

>>11322736
No, it's full size.
Good point tho, might be better to link to https://i.imgur.com/vPAp2YD.png

>> No.11322743
File: 59 KB, 338x171, 1574041756645.png [View same] [iqdb] [saucenao] [google]
11322743

Apologies if this has been asked a lot, I've googled a bit already and I don't frequent /sci/ ever.
Is there any difference between the various ancestry DNA tests? I'm looking to upload the results to third party websites as well, are some of them more in depth? For example: I see that FamilyTreeDNA does Y Haplogroup analysis kits specifically, I take it 23andMe does not sequence it as in-depth as FamilyTreeDNA would?

>> No.11322764

>>11322653
its perfect, good job

>> No.11322997

>>11314509
To flip a square matrix Aij about the main diagonal, you just switch the i and j to get the transpose, right? So, the matrix flipped about the main diagonal would be Aji.

What do you do to flip a matrix about the /other/ diagonal?

>> No.11323295

>>11321863
Did you go to the doctor?

>> No.11323389
File: 43 KB, 690x774, 555.png [View same] [iqdb] [saucenao] [google]
11323389

What should you proof/discover/create in fields of mathematics or physics today to make a contribution that will be known by average person in 100 years?

>> No.11323392
File: 79 KB, 923x713, 1528873732679.jpg [View same] [iqdb] [saucenao] [google]
11323392

how would a working quantum computer change our society? when i searched for answers on (you)tube i only find I LOVE SCIENCE videos

>> No.11323410

>>11322997
To get the transpose B of A, you let Bij=Aji
To get the reflection over that other diagonal, you let Bij=Aki where k=n-j+1 for an n×n matrix
(I think)

>> No.11323412
File: 162 KB, 400x283, article-g01_400_283.jpg [View same] [iqdb] [saucenao] [google]
11323412

Could the widespread use of the C57BL mice (and similar) be harmful to the data analysis of a study? Won't using only clones harm the external validity of the study?

>> No.11323420

>>11323389
Unify the fundamental forces, discover a fundamental particle not known yet, or prove P ?= NP.
Note that your discovery will not be truly "known" by the average person but they might recognize you or a gross oversimplification of your work from a cartoon. So much physics was discovered in the early-mid 20th century but mostly people just remember Einstein's face and E=mc^2 (without any appreciation of what that means).

>> No.11323438
File: 193 KB, 1643x1200, x7.jpg [View same] [iqdb] [saucenao] [google]
11323438

>>11323389
Assuming you weren't a wunderkind like Tao, the best strategy is probably immitating Feynman:
>make solid, relevant contributions to your field
>set up some drama like adulterous relations, stealing results
>also random stuff that has shitpost potential, like the IQ thing
>publish good expositional material
It's extremely hard, sure, but it's probably quite a bit easier than making Eintein-tier contributions to your field.

>> No.11323440

>>11323438
Feynman's genius was in simplifying complex ideas

>> No.11323445
File: 225 KB, 690x386, x12.jpg [View same] [iqdb] [saucenao] [google]
11323445

>>11323440
Feynman's genius was *also* simplifying complex ideas.

>> No.11323449

>>11323445
he couldn't even explain magnets in simple terms. just got mad and said you don't know what you are asking. if you can't explain something in simple terms that means you don't understand it.
https://www.youtube.com/watch?v=MO0r930Sn_8

>> No.11323462

>>11323449
Im pretty sure Feynman, the guy who helped make QED, understood something or another about magnets. Do you?

>> No.11323474

>>11323438
>publish good expositional material
Very underrated how good of an idea this is. Successful textbook writers get far more well-known than all but the top handful of researchers. Everybody knows the names of people like Stewart and Gallian despite them producing quite little original work of quite mediocre quality, and the same pattern holds true even for people who did do really good work (I'd wager that far more people know about Atiyah-Macdonald than could tell you what a Macdonald polynomial is).

>> No.11323512
File: 116 KB, 957x819, I HATE THAT HEDGEHOG.png [View same] [iqdb] [saucenao] [google]
11323512

Why the fuck do scientists think a drug named after Robotnik will be able to inhibit the Sonic Hedgehog signaling pathway? Has Robotnik ever successfully stopped Sonic? Are scientists not real gamers?

>> No.11323522

Why does the mirror mix up left and right but not the up and down?

>> No.11323531

>>11323522
Mirrors flip forward and back, along the axis normal their surface. Not left to right, not up and down.

>> No.11323616

>>11323531
don't we see upside down and our brain then flips the image? also, how do we know what the mirror sees? or for example a film camera. does it capture the flipped image or the real image? then when film is printed, is it flipped again? what is the "real" image even, is it flipped originally?

>> No.11323618

>>11323616
What your brain is doing isn't really relevant to any of this. SLR cameras do indeed flip the image (as does you brain).

>> No.11323630

>>11322997
anon...

>> No.11323703

how do we know bad taste or bad smell from a good one? are we born with this or thats an acquired taste? say farts vs fragrance. if we were exposed to farts used as fragrance would we grow up thinking it is a good smell?

>> No.11323719

How do I prove that if a polynomial p(x) satisfies p(x)=p(x+1) then it's a constant?

>> No.11323740
File: 55 KB, 680x695, 1579376777933.jpg [View same] [iqdb] [saucenao] [google]
11323740

>>11321863
man up

>> No.11323749

How can light be both discrete and continuous?

>> No.11323762
File: 120 KB, 411x712, x15.jpg [View same] [iqdb] [saucenao] [google]
11323762

>>11323719
Set [math]f(x)=p(x)-p(0)[/math].
Then [math]f(k)=0[/math], for any integer [math]k[/math].
But a polynomial can't have an infinite amount of zeroes, so it's constant.

>> No.11323859
File: 42 KB, 928x386, you_what_mate.png [View same] [iqdb] [saucenao] [google]
11323859

>>11314509
How am I meant to interpret this question? Does v form a row vector of a 1x4 matrix? I have done these calculations before using numbers in a matrix but I have never done it before with variables like this. Does this mean that v_1, v_2 etc are just components of the vector v?

>> No.11323862

What enforces the Pauli exclusion principle, in the sense that what force/mechanism prevents shoving extra electrons into a quantum state that's already occupied, and how is this overcome in the formation of a black hole?

>> No.11323884

>>11323392
for one it would fuck up the current way we're doing public encryption. I think there are other ways to do it so we might still be fine.

>> No.11323890

Why doesn't anybody ever talk about time-loop logic(Novikov Self-Consistency)? People talk about other forms of time travel all the time, but this is the most practical application of such a thing. If you can send even a tiny amount of information back a short amount of time, you can use it to solve NP Complete problems and more very quickly.

>> No.11323894

>>11322183
>8-12 hours
?

>> No.11323901

>>11321863
it probably doesn't help you to just stay inside and not have contact with people, I suggest going out even if you don't like it, maybe go for a walk. Also physical exercise can help too, it won't be instantaneous but you should start to see results after a while, also stop jerking off

>> No.11323904

>>11323862
Apparently cooper pairs can condense like bosons, it just takes tremendous energy.
A lot of quantum mechanical "rules" have loopholes. Only catch is they take ridiculous amounts of time and energy

>> No.11323939
File: 111 KB, 937x264, V11.jpg [View same] [iqdb] [saucenao] [google]
11323939

>>11323859
Please try to sound less confused, it completely kills my motivation to answer the question.
Anyhow, v is a collumn vector (as seen on the right).
>Does this mean that v_1, v_2 etc are just components of the vector v?
Yes.

>> No.11324098

>>11323522
Perception. We perceive it as being a horizontal flip because we perceive space as having a vertical axis and a horizontal plane.

Consider a mirror whose normal lies in the horizontal plane. Construct a 3D coordinate system with orthogonal axes N (the normal to the mirror), V (the vertical) and H (the axis which lies in both the plane of the mirror and the horizontal plane).

A reflection in N is equivalent to a rotation about V followed by a reflection in H. It is *also* equivalent to a rotation about H followed by a reflection in V. We perceive it as the former rather than the latter. That is all.

>> No.11324114

>>11324098
So wait a second, is this also a Dihedral group? D4, if we have a square mirror?

>> No.11324147

>>11324114
No.

Also: more generally, a reflection in N can be decomposed into a rotation about an arbitrary vector X and a reflection in N×X.

Ultimately, the idea that a mirror flips "left to right" is because we compare it to a rotation about the vertical rather than about some other axis.

I'm assuming that we assume rotation about the vertical because that's what we're used to. A rotation about the vertical keeps us "upright", which is how we normally function. Creatures which don't particularly care about orientation (e.g. spiders) might perceive it differently.

>> No.11324189

why is djvu such a shitty format? i've never seen a good quality djvu book on libgen AT ALL, they are all shitty scans. very blurry compared to most pdfs except maybe for very old pdf scans, those are horrible too.

>> No.11324248
File: 1.05 MB, 764x1046, yukari13.png [View same] [iqdb] [saucenao] [google]
11324248

>>11323392
It will shift complexity space around a little, but certain problems will remain NP. The boldest claim I've heard is from Nayak who says that the traveling salesman will become P with a quantum computer. Other than that it'll just be the issue of teaching people how to design and implement quantum algorithms.
>>11323884
I doubt that, people have already been developing quantum encryption algorithms since the 80's.
>>11323904
In BCS theory, Cooper pairs form between electron states lying at diameterically opposite ends of the Fermi surface, so their states are distinct and aren't subject to Pauli exclusion. Even non-BCS (e.g. FFLO) SC have Cooper pairs labeled by different bands; no violation of Pauli exclusion here at all, and no violation of that will ever occur.
>tremendous energy
Quite the opposite my dear, it takes extremely low energy and temperatures; BCS theory is a weak-coupling theory between electrons due to an effective attraction, typically an electron-phonon interaction. This requires a very "still" tight-binding configuration of the lattice which is easily destroyed by thermal fluxtuations. Once Coulomb takes over the electrons are never getting back.

>> No.11324576

>>11321756
Bamp

>> No.11324658
File: 2.68 MB, 4000x3000, crystals.jpg [View same] [iqdb] [saucenao] [google]
11324658

I'm freeze distilling applejack and this is round 4, how come the liquid that doesn't freeze is forming crystal sheets?

>> No.11324801

Why is 13 - 8 + 4 equal to 9 and not 1?
PEMDAS puts Addition before Subtraction so shouldn't be 13 - 12 = 1?

>> No.11324806
File: 2.13 MB, 984x1280, boozebrain.png [View same] [iqdb] [saucenao] [google]
11324806

>>11324658
no idea,
neat-o crystal sheets, neat-o project

>> No.11324912

>>11324801
mathematical laws

>> No.11324919
File: 137 KB, 500x500, __remilia_scarlet_touhou_drawn_by_sindre__6c45a4d2c1f78b2f7481e43ead082f41.jpg [View same] [iqdb] [saucenao] [google]
11324919

>>11324801
>PEMDAS puts Addition before Subtraction
PEMDoesn't.
https://www.mathsisfun.com/operation-order-pemdas.html

>> No.11325163
File: 28 KB, 982x414, solid pulley.png [View same] [iqdb] [saucenao] [google]
11325163

>>11314509
Hi can you help me? the result i found is nonsense cause the acceleration is grater than gravity's. and i cant find any mistake at all.

>> No.11325228

>>11324806
how is drinking alcohol while pregnant even legal much less socially acceptable?

>> No.11325254

>>11325163
I'm 100% sure you made a silly mistake. Post your work.

>> No.11325268

>>11325228
It isn't. What the fuck country are you from where it is?

>> No.11325298
File: 7 KB, 240x235, IMG_0467.jpg [View same] [iqdb] [saucenao] [google]
11325298

How come I can jump on something like this all day and all directions with 130 kilograms and it flies apart when you smack it with a 12 ounce hammer?

>> No.11325309

>>11325254
torque=0,2*g*(0,6-0,05(0,2*cos30+sen30))=0,2*(weigh3 - tangent weight1 - friction1)
moment of inertia=0,4*(0,2)^2/2
aceleration=0,2*torque/moment=(2/0,4)*g*(0,6-0,05(0,2*cos30+sen30))=2,83g

>> No.11325315

>>11324658
Might be sugar, taste it

>> No.11325390

Skin absorbs water in pours so after taking a shower how long would it take for the water to go from the pours of hands to the bloodstream or wherever it goes or in other words how long does it take for the hands to be fully dry?

>> No.11325517
File: 6 KB, 424x103, Screenshot_2020-01-23_13-34-28.png [View same] [iqdb] [saucenao] [google]
11325517

i jsut cant do it

>> No.11325522

>>11325517
wait i did it right after posting this
its 14 and 21..

>> No.11325562 [DELETED] 
File: 441 KB, 370x554, pinky.png [View same] [iqdb] [saucenao] [google]
11325562

>>11325309
>>11325163
I can't be arsed to go through your calculation, but the most important thing to realize about this problem is (1) the acceleration of each block is the same and (2) the linear acceleration of the blocks is just the radius of the pulley times its angular acceleration.
First, sum the torques on the pulley:
[eqn] \sum\tau=I\dot\omega\implies T_1-T_3+\frac{1}{2}m_pa=0 [/eqn]
Then, block 1:
[eqn]\sum F=m_1a\implies T_1-m_1a=m_1g(\sin\theta-\mu\cos\theta)[/eqn]
Then, block 3:
[eqn] \sum F=m_3a\implies T_3+m_3a=m_3g [/eqn]
So you have three linear equations three unknowns. You may easily solve for both tensions and acceleration at this point.
[eqn] a=\frac{m_3g+m_1g(\mu\cos\theta-\sin\theta)}{m_1+m_3+m_p/2} [/eqn]
>>11325298
When you hit the thing with a hammer, there is an extreme concentration of stress right around where the hammer impacts, much much greater than if you where to stand or even jump. Stress concentrations are the enemy of built-up components, like a few two by fours fastened with some nails. Also, the hammer is putting a lot more than 12 oz of force on the thing (something closer to about 100 lbs, from google) depending on how fast you swing it, how hard the wood is, etc.
>>11325390
>pours
>>11325517
good boy

>> No.11325608
File: 441 KB, 370x554, pinky.png [View same] [iqdb] [saucenao] [google]
11325608

>>11325309
>>11325163
I can't be arsed to go through your calculation, but the most important thing to realize about this problem is (1) the acceleration of each block is the same and (2) the linear acceleration of the blocks is just the radius of the pulley times its angular acceleration.
First, sum the torques on the pulley:
[eqn] \sum\tau=I\dot\omega\implies T_1-T_3+\frac{1}{2}m_pa=0 [/eqn]
Then, block 1:
[eqn]\sum F=m_1a\implies T_1-m_1a=m_1g(\sin\theta-\mu\cos\theta)[/eqn]
Then, block 3:
[eqn] \sum F=m_3a\implies T_3+m_3a=m_3g [/eqn]
So you have three linear equations with three unknowns. You may easily solve for both tensions and acceleration at this point.
[eqn] a=\frac{m_3g+m_1g(\mu\cos\theta-\sin\theta)}{m_1+m_3+m_p/2} [/eqn]
>>11325298
When you hit the thing with a hammer, there is an extreme concentration of stress right around where the hammer impacts, much much greater than if you where to stand or even jump. Stress concentrations are the enemy of built-up components, like a few two by fours fastened with some nails. Also, the hammer is putting a lot more than 12 oz of force on the thing (something closer to about 100 lbs, from google) depending on how fast you swing it, how hard the wood is, etc.
>>11325390
>pours
Anon, your hands dry because the water evaporates, not because it is going into your blood stream. What the fuck
>>11325517
>>11325522
good boy~

>> No.11325615

>>11325268
As far as I know, it's not illegal in the uk and it doesn't seem to be illegal in new york either. Also, re prevalence:
https://bmjopen.bmj.com/content/5/7/e006323.full

>> No.11325621

>>11325615
>legal
I misread. Yeah, it's not illegal. But it is by no means socially acceptable.

>> No.11325627
File: 93 KB, 801x366, Screen Shot 2020-01-23 at 11.23.22 AM.png [View same] [iqdb] [saucenao] [google]
11325627

I am not able to go from the second to last line, to the last line.

>> No.11325629

>>11325621
Bruh I just showed you how prevalent it is, while sure this maybe means that they all do it in secret, I think it's possible that people would just be lenient about it, I even saw a uni girl smoking weed while pregnant.

>> No.11325636

>>11325627
Have you tried just expanding it out?

>> No.11325642

>>11325629
>Australia, UK, Ireland
Ah, no wonder they are such genetic catastrophes

>> No.11325645
File: 45 KB, 320x322, Capture.png [View same] [iqdb] [saucenao] [google]
11325645

>>11325627
Did you try expanding it by hand and seeing if things cancel out?
>are you serious
Unfortunately.

>> No.11325764

>>11325522>>11325517
dumbass. just realize you are summing an area. and then removing part of it. the remaining is the rest. (like cuting a piece of cake)
>>11325562
what is "Mp*a/2" and where did that ecuation came from? why are you not multipling the forces by the distance? Anyways i see where i fucked up, and can solve it properly now. i just asumed the tentions where the opposite as he sum of the oher forces. Big brainlet cause the acceleration is NOT 0.

>> No.11325766

>>11325764
>what is "Mp*a/2" and where did that ecuation came from? why are you not multipling the forces by the distance? Anyways i see where i fucked up, and can solve it properly now. i just asumed the tentions where the opposite as he sum of the oher forces. Big brainlet cause the acceleration is NOT 0.
meant to>>11325608

>> No.11325812

>>11314509
>/qtddtot/
what is that? btw /sqt/ is a horrible name too. it should be something easily recognizable, not some acronym. what not just /stupid/

>> No.11325838
File: 168 KB, 771x807, 1574217774251.jpg [View same] [iqdb] [saucenao] [google]
11325838

>>11325764
>>11325766
>calls anon a dumbass
>is a dumbass himself
I summed the torques, which would each be tension*radius, but the radius is a common factor which divides out. Do the algebra and see for yourself.
>>11325812
/questions that dont deserve their own thread/
/sqt/ and /qtddtot/ are good names because that's what the stupid question threads on other boards like /g/ and /diy/ are called

>> No.11325841

>>11325608
>hammer
So, pressure? My foot is about 48 square inches and my hammer is one. This seems the opposite effect of how a hydraulic jack works but there is a speed trade off. I can handle a little math. pfa, fma, psi. If you take it easy. I don't understand how hitting stuff works. Is it the same kinda deal when a bullet penetrates an engine? Is there a book for simpletons?

>> No.11325861

>>11325812
>acronym
I want to express my wish to resist the common usage of this word. Acronyms should be pronounceable. Those are initials. I'm wrong but I don't want to be. It is how it was in the past.

>> No.11325871

>>11325861
I don't care about the exact semantics but I am fine with this definition as long as threads like this have pronounceable names. /stuq/ for example.

>> No.11325894

>>11314509
Is there a patrickjmt equivalent for physics? He saved me for calc 1. I'm doing university mechanics as an elective and would like resources to help me get a good grade despite being a non-science major.

>> No.11325919

>>11315890
You could commision a bigger one, put some padding around, a seat and barrier to keep you inside and done

>> No.11325921
File: 97 KB, 413x413, 20190413_062333.jpg [View same] [iqdb] [saucenao] [google]
11325921

>>11325841
>pressure
No, stress. It's like pressure and has the same units, but it is a tensor quantity. In the case of impact loading, like when you jump on the thing or hit it with a hammer, the force of impact is heavily dependent on the material's elastic properties. The more an object deforms, the less force there is. An exact description of what happens when objects collide is fairly complicated. Look up "coefficient of resititution". The theory of elasticity can be as complicated as you like.
>book
I like Mechanics of Materials by Beer.
Roark's Formulas for Stress and Strain is an absolute classic engie text, good source for stress concentrations and all that.
Also, continuummechanics.org for a more detailed description of elasticity. This is a great source because it builds up all the math as it goes along.

>> No.11325939

>>11325636
>>11325645
I did, yeah. On my first go around I used the wrong value for "the sum of n^3" though. and after that I was nearly out of time (had to do other things) and a little impatient.

I'll try again later, was hoping someone would just type of some TeX so I could quickly find the step I was either missing or fucking up.

>> No.11325968
File: 121 KB, 1000x1000, __remilia_scarlet_touhou_drawn_by_batta_ijigen_debris__3b98af08e08b5c35fdd64f81ba720ed7.jpg [View same] [iqdb] [saucenao] [google]
11325968

>>11325812
For years now, we've been using /sqt/ (pronounced es-qe-tit) in remembrance of our great friend, Lil Pump, who used to browse this place and assist people with their problems. I still remember how he saved me when I had that extremely hard test on Kodaira-Spencer's deformations of complex manifolds with professor Misha.
>what not just /stupid/
Because that's fucking retarded.
>>11325861
>>11325871
>esketit isn't pronounceable

>> No.11326041
File: 6 KB, 560x568, Untitled.png [View same] [iqdb] [saucenao] [google]
11326041

How does complex integration works? Is there any relationship between the colored parts? I would want to think the blue and green would add up to the yellow or yellow+pink, but the more I think about it this is bullshit. Since sin can be any frequency, but the frequency is not accounted for on the unit circle. But it is almost correct for w=1: and angle=PI/2: cos from 0 to PI/2 = 1. And 1/4 of the unit circle area is PI/4 which is 0.78539816339 so almost 1. I may be onto something. I am not looking this up on purpose: I am rediscovering sciences for myself based on the intuition. Can you give me a push in the right direction?

>> No.11326043
File: 6 KB, 560x568, Untitled.png [View same] [iqdb] [saucenao] [google]
11326043

>>11326041
wtf where is my pink??? let me try to sink the pink again.

>> No.11326144
File: 201 KB, 636x773, owari da.png [View same] [iqdb] [saucenao] [google]
11326144

>>11314509
what is the best way to study im just starting medschool and im scared im wasting my time just reading also im pretty low iq desu ive always managed with pure effort but can i still make it on this level on effort alone

>> No.11326151
File: 38 KB, 403x433, be patient i have autism.jpg [View same] [iqdb] [saucenao] [google]
11326151

will an infimum always exist if there is a lower bound? If it can be proven that a set has some lower bound, will it always have an infimum, if so how do you prove it? Sorry if this is a stupid question but i can't seem to find out how to prove this.

>> No.11326193

>>11326151
>will an infimum always exist if there is a lower bound?
Yes, as long as the set is non-empty.
>if so how do you prove it?
For what construction of the reals?

>> No.11326270

>>11326193
>For what construction of the reals?
Sorry, what do you mean by that? All i can say is that the set is just all terms in a sequence.

>> No.11326303
File: 956 KB, 1125x1607, skates.jpg [View same] [iqdb] [saucenao] [google]
11326303

>>11326270
Ah, I see.
We call the set [math]S \subset \mathbb{R}[/math]. Since [math]S[/math] is nonempty, [math]a \in S[/math]. [math]S[/math] also has a lower bound [math]b \in \mathbb{R}[/math].
We consider [math]a', ~ b'[/math] to be the integer parts of [math]a[/math] and [math]b[/math].
There are a finite number of integers between [math]a'[/math] and [math]b'[/math]. We pick the largest one which is also a lower bound, [math]c[/math].
Now, we pick the largest [math]n \in \mathbb{N}[/math] such that [math]c.n[/math] is also a lower bound, where n can be zero.
By iterating this procedure infinitely for the rest of the decimal cases, we produce a number [math]i[/math], which is the infima.
First, if [math]d \in S[/math] and [math]d<i[/math], then it is larger in some decimal case. This can't happen because of the construction. Thus, [math]i[/math] is a lower bound.
If some other [math]e[/math] is larger than [math]i[/math], then it is larger in some decimal case. Thus, by construction, it isn't a lower bound.

>> No.11326306

>>11326144
not sure if bait because I'm a bit autistic, but
medschool = memorization
memorization = skill

>> No.11326315

>>11326303
Small mistake: I assumed [math]c[/math] is positive.
Correcting this is left to the reader as an exercise.

>> No.11326324

>>11326303
oh ok thanks. So from my understanding you basically start from the floor of the smallest element and keep adding decimal places to get closer to the actual value. However would this still be considered the infima as you only approach the value?

>> No.11326353

>>11326324
>from the floor of the smallest element
No, not at all. There isn't necessarily a smallest element.
>However would this still be considered the infima as you only approach the value?
I'm considering a real number to be an infinitely long decimal number. All the decimal cases of i are given, and thus it's well defined.
This all works out formally, even if it isn't the usual way to formally do this. That's why I asked earlier about which construction you were thinking of.

>> No.11326358
File: 601 KB, 680x680, tired.png [View same] [iqdb] [saucenao] [google]
11326358

>>11326306
WHAT THE FUCK DOES THAT MEAN ANON

>> No.11326377

>>11326353
>There isn't necessarily a smallest element.
I forgot to mention this and it relates to what you just said. So how would you know that $c.n$ would still be lower bound if you don't know precisely any of the elements, especially if there are say an infinite amount of elements and no smallest element?
>I'm considering a real number to be an infinitely long decimal number
after looking at the epsilon definition of an infima i guess the infinite decimal still meets the definition.

>> No.11326391

>>11326377
n is an integer between 0 and 9.
I merely take the subset of those such that c.n is a lower bound, and pick the largest n.
I don't need to worry about infinities. I'm picking the largest number in a finite set.

>> No.11326412

>>11326391
ok thanks. also how do you use latex om 4chins? Is it also just a /sci/ thing?

>> No.11326447

>>11326412
>ok thanks. also how do you use latex om 4chins? Is it also just a /sci/ thing?
Refer to https://i.imgur.com/vPAp2YD.png
It's just /sci/.

>> No.11326518

>>11326041
> How does complex integration works?
Are you talking about integrating a function in R->C or C->C?

For the former, you can just integrate the real and complex parts separately. For the latter, it's a line integral; but if the function is holomorphic everywhere, then the value of the integral depends only upon the endpoints and is independent of the path. More generally, two line integrals with the same endpoints have the same value if the function is holomorphic everywhere in the region between the two lines. A corollary of this is that an integral around a closed loop is zero if the function is holomorphic everywhere inside the loop.

>> No.11326565

>>11325871
I don't make the rules friend. I can relate though. A word as you suggest is "portmanteau." A combination of words for another word. Like Pokémon is "pocket monster."
>>11325968
Thanks for the history. Not the same as the rapper. Is it?
>>11325921
You have made the world a less stupid place. Noted and appreciated.

>> No.11326572
File: 162 KB, 500x464, 1501333971240.png [View same] [iqdb] [saucenao] [google]
11326572

>>11326041
Given a vector bundle [math]p:E\rightarrow X[/math] of rank [math]k[/math], you can perform integration on forms [math]\alpha\in\Omega^p(X)[/math] along the fibres as [math]p_*\alpha(x) = \int_{E_x}(\operatorname{ev}^*_{p-k} \alpha)d\operatorname{vol} [/math]. We usually do this on the sphere bundle [math]\mathcal{S}(E)[/math] or the Thom space [math]X^p[/math] for their compactness, which lets us use Stokes's without worrying about boundary terms.
Now if [math]E[/math] has a holomorphic structure such that we can perform Hodge decomposition [math]\Omega^n = \bigoplus_{p+q=n}\Omega^{p,q}[/math], we need fibre integration [math]p_*[/math] to be compatible with this decomposition as well; namely [math]p_*\alpha^n = \sum_{p+q=n} p_*\alpha^{p,q}[/math], which allows us to use holomorphic Stokes's as well as tools from Dolbeault cohomology. This is the essence of complex integration when looking at [math]n=0,1[/math]-forms.

>> No.11326575

>>11314509
A friend taking his first physics class asked my help with the following problem, did I solve it correctly?

>Let [math]\vec{B} = 6i - 8j[/math]
>[math]\vec{C} = \vec{A} + \vec{B}[/math] where [math]||\vec{C}|| = ||\vec{A}||[/math] and [math]\vec{C}[/math] has a direction lying solely towards the x axis (it has no y component)
>Find [math]||\vec{A}||[/math]

How I solved it:

[eqn]\vec{C} = (a_x + 6)i + (a_y - 8)j[/eqn]

Since C doesnt have a y component:

[eqn]a_y = 8[/eqn]

And since the norm of A is the norm of C:

[eqn]\sqrt{(a_x + 6)^2} = \sqrt{a_x^2 + a_y^2} \\ a_x^2 + 12a_x + 36 = a_x^2 + 64 \\ a_x = \frac{64 - 36}{12} = 7/3 \square[/eqn]

Now if im wrong please point where, its been ages since ive done shit like this.

>> No.11326576
File: 6 KB, 553x527, Untitled.png [View same] [iqdb] [saucenao] [google]
11326576

>>11326518
R->C, like e^iwt. i get that it is the same as cos(wt)+isin(wt), but I am not sure how to visually show that the area on the circle is the sum of sin and cos so it would match two corresponding areas for cos and sin for a given interval. I am talking about this picture

>> No.11326593
File: 72 KB, 633x758, 1531077591342.jpg [View same] [iqdb] [saucenao] [google]
11326593

>>11323295
Yes, I've been diagnosed with major depression, general anxiety disorder, social anxiety disorder, ptsd and agoraphobia.
I've been going to therapy since I was a kid (I was raped when I was 5yo) and I went to a psychiatrist for the first time at 16yo but I refused to take meds and I mostly lied to him. Second time with a psychiatrist I was 18yo, they told me that SSRI were the solution and I was really bad at the moment so I accepted, but I actually got worse. Stop taking them a year and a half after that. Then I went to another psychiatrist and I refused meds but since I have a lot of panic attacks I accept clonazepam, but I only take it during a panic attack or when I think I'm about to have one, etc (less than twice a month nowadays). My new therapist thinks that I should go to the psychiatrist and I should take antipsychotics, but that shit makes you lazy, fat and stupid and I don't want that. I can study but not enough, I usually start having self harming thoughs when I try to concentrate and end up loosing my concentration. Maybe Adderall could help me with that, I just wanted to hear some experiences.
>>11323740
I wish I could but I'm a retarded autistic faggot.
>>11323901
I had never jerked off nor watched porn. I can't enjoy things and I don't have any libido. I'm currently in a farm going out everyday, it's easy for me to go out when there's no people around. I have exercise a lot (between 3 and 12 hours per week) almost my whole life, otherwise I can't sleep. The thing it's that it's really hard for me to go out in the city where I study cause it's a dangerous city, it's CROWDED af, and most of my classmates lives really close to me and I'm afraid of them due to some trauma.
I also tried meditation but it's currently kind of hard for me rn cause when I try not to think I start crying without reason and sometimes I start having self harming thoughts, but I'm still trying.

>> No.11326634

>>11326575
Looks good.
>>11326593
>I just wanted to hear some experiences.
I'd love to talk with you about experiences, anon, and I really hope it gets better. But I'm not sure if this board let alone this thread is an appropriate place.
>>11326572
As far as strategies for responding to semi-literate or schizo posts go, this is excellent. Highly based.
>>11326565
You are very kind~

>> No.11327212

>>11326634
>Looks good
Seriously? Apparently the question was multiple choice and none of the choices were 7/3. I do believe my resoning is correct and I don't see any errors in ny algebra

>> No.11327267

Planning to complete all 14 chapters of cengal's heat transfer book in the next 48 hours. Is this possible?

>> No.11327344

>>11327212
Yep. I don't know what to tell you, unless I am also blind to some simple error.
>>11327267
No

>> No.11327475

[math] LaTeX [/math] question here:
I'm making a presentation in beamer and on one frame I want to make an itemize list in a left column with \pause in between every item.
On the right column I want to show a graphic for every item that gets replaced by another graphic when another item comes up.
Anyone has an idea how to do this?
One obvious way would be to just clone the frame into many frames with different graphics but then the itemize block would be shifted around with every slide change.

Maybe it's possible to set a fixed vertical position for the text to start on the left column?
thanks in advance.

>> No.11327477

>>11327475
FUCK I meant to type [math] \LaTeX [/math]

>> No.11327537

>>11327344
Seems the options where wrong then.
Thanks for the help

>> No.11327884 [DELETED] 
File: 57 KB, 530x397, Untitled2.png [View same] [iqdb] [saucenao] [google]
11327884

How do I geometrically express integration on the unit circle from 0 to PI/2
in terms of the area under the curve for [math]\int_{0}^{\pi/2}e^{ix}dx[/math]?
Should the yellow area be equal the sum of the blue and green? It is not.
Both integrals for sin and cos = 1, so the result is 1+i which has the magnitude of sqrt(2). But the area of 1/4 of the unit circle is PI/4. What gives?

>> No.11327996

why are all internal combustion engine cars being phased out in favor of electric cars, what about biofuels

>> No.11328002

>>11327996
>why are all internal combustion engine cars being phased out
This is definitely not true

>> No.11328009

>>11328002
but everyone is acting like electric cars are definitely the future even though they use lithium and neodymium etc. some countries are banning gas and diesel cars but biodiesel can be used with standard diesel engines so just convert the gas stations

>> No.11328012

>>11328009
>everyone is acting
More hyperboles. Come back when you have an actual question.

>> No.11328017

>>11328012
oh you know what i mean. every libtard tech youtuber and james may etc, everyone like that is shilling electric cars

>> No.11328023

>>11328017
So your question is why do libtard tech youtubers act a certain way? I don't know. Not sure if this is the board for that.

>> No.11328030

>>11327996
>why are all internal combustion engine cars being phased out in favor of electric cars

... because we can make zero carbon electricity, but zero carbon biofuels aren't really a thing. Carbon per kilowatt is a declining metric that can cross zero. Carbon per gallon isn't.

(tinfoil hat mode engaged)
Additionally, the ability to stop transportation by cutting off power provides an additional control for those in power in the event of a societal breakdown.

>> No.11328033

>>11328023
a lot of countries are discouraging the use of ICE cars with high fuel taxes and some are even banning the sale of ICE cars outright, including diesel cars which can use biodiesel with no need to modify the engine, so what gives, why not invest in things like biodiesel instead of forcing the EV meme like norway for example is doing

https://en.wikipedia.org/wiki/Yellow_vests_movement
https://en.wikipedia.org/wiki/Phase-out_of_fossil_fuel_vehicles
https://elbil.no/english/norwegian-ev-policy/

>> No.11328040

>>11328033
>so what gives, why not invest in things like biodiesel instead of forcing the EV meme like norway for example is doing
Because EV has much greater potential than biodiesel. There is much research being done on powerstorage. Why are you so staunchly committed to a century old technology?

>> No.11328050

>>11328040
>Because EV has much greater potential than biodiesel.
[citation needed]

>> No.11328053

>>11328050
This the the perception and opinion of government officials. I don't have a source, but it shouldn't really sound controversial either. You asked for the reason. This is the reason, in my opinion.

>> No.11328106
File: 107 KB, 700x734, 558.jpg [View same] [iqdb] [saucenao] [google]
11328106

>>11328053
>muh opinion
>muh feels
>it shouldn't really sound controversial

>> No.11328112
File: 48 KB, 961x433, Screenshot_20200124_125748.png [View same] [iqdb] [saucenao] [google]
11328112

why is geometry so unintuitive for me

help

>> No.11328123

>>11328106
Do you disagree?

>> No.11328124

>>11323762
why can't a polynomial have an infinite number of zeroes?

>> No.11328132
File: 87 KB, 960x640, https___specials-images.forbesimg.com_dam_imageserve_42963422_960x0.jpg_fit=scale.jpg [View same] [iqdb] [saucenao] [google]
11328132

>>11328123
the best commercial EV technology we have right now relies on heavily polluting mining practices in third world countries
we don't know if EVs will win out in the long run or if the electricity is better used in the production of hydrogen fuel or something else instead of having a huge ass battery in every car

>> No.11328156

>>11328124
it can

>> No.11328158

>>11328132
Okay. And yet the public has a different perception.

>> No.11328162

>>11328124
It can, but then it's constant.
Or you're working in a very shitty ring.

>> No.11328164

>>11328162
>what is cos(x)

>> No.11328166

>>11328164
>cos(x) is a polynomial

>> No.11328168

>>11328158
things change
not long ago, this was the agenda, anyone could enter womens bathrooms etc
https://www.youtube.com/watch?v=VtJFb_P2j48

>> No.11328171

>>11328166
Is a taylor series not a polynomial?

>> No.11328173

>>11328171
No, it isn't.
A power series is a power series, a polynomial is a polynomial.

>> No.11328222

help a brainlet please

what's a better deal

70% alcohol by volume for $109
or
83% alcohol by volume for $119.98

>> No.11328236
File: 19 KB, 400x400, pm7tsm4njtu01.jpg [View same] [iqdb] [saucenao] [google]
11328236

>>11325627
BUMP PLEASE HELP

I don't understand how the n^3 term simplifies to n^3/3, I continually keep getting 2n^3/3 and see no other way.

the (n+1)^5 expands into a polynomial containing 2n^3, the (10*(n^3/2))/5 simplifies to -n^3, and the n(n+1)(n+2) expression contains -n^3/3 after simplification. So 2n^3 - n^3 -n^3/3 = 2n^3/3, but obviously I'm wrong.

>> No.11328263

>>11328236
Also, in the final expression, instead of -n/30 I keep getting -11n/30 :( I kind of hope it's errata on the author's part, but I doubt it.

>> No.11328265
File: 1.24 MB, 947x1316, __flandre_scarlet_and_remilia_scarlet_touhou_drawn_by_sukocchi__12c176b09b4f922070610155ad7231aa.jpg [View same] [iqdb] [saucenao] [google]
11328265

>>11328236
My mood was absolutely excellent, so I typed it out.
(n+1)^5 -1 - 10(n^4 /4 + n^3 /2 + n^2 /4) - 10(n(n+1)(n+2))/6 -5(n(n+1)/2) - n [Note that the /5 was left out.]
Just copy and paste it into Wolfram. You'll get a slightly different final form.
Might be a typo in one of the steps.

>> No.11328271

>>11328265
>(n+1)^5 -1 - 10(n^4 /4 + n^3 /2 + n^2 /4) - 10(n(n+1)(n+2))/6 -5(n(n+1)/2) - n
Thanks for your kindness, I forgot about things like Wolfram for these little headaches.

>> No.11328279

>>11328265
>(n+1)^5 -1 - 10(n^4 /4 + n^3 /2 + n^2 /4) - 10(n(n+1)(n+2))/6 -5(n(n+1)/2) - n
Well, what the fuck, wolfram agrees with me? Would you mind taking a second look at this anon?

>> No.11328290

>>11328279
That's why I said it might be a typo.

>> No.11328297

>>11328290
Well, I can't find it if is. I'd like some sort of external confirmation before moving on, but I know it's tedious. I've checked that input nearly a dozen times now, and even manually reentered it myself a couple times.

>tfw no errata for the 3rd ed of Spivak's Calculus, only the fourth ed

>> No.11328311

I'm asked to solve this differential equation
u(t) = Ri + L di/dt
where u(t) is a unit step function.
Any ideas on how to do this? It's been like 2 years since I've taken DE.

>> No.11328323
File: 234 KB, 600x665, 3375818f8594c3b469fe22180b5eb6dd.png [View same] [iqdb] [saucenao] [google]
11328323

>>11328311
Use a Laplace transform. Recall that L(u)=1/s and L(i')=si-i0

>> No.11328352

>>11328297
>>11328279
>>11328236
>>11325627
It would be really nice to close this open loop this afternoon, just clarifying I'm still unsure if me + Wolfram are right and Spivak is wrong, or if I entered it wrong/fucked up somehow.

>> No.11328368

just a random thought: can strange attractors be used in sound synthesis? would be cool to listen to them.

>> No.11328391

>>11325627
>>11328352
The formula substituted for the sum of squares in the solution is wrong. [math]\sum_{k=1}^{n}k^2 = \frac{n(n+1)(2n+1)}{6}[/math]

>> No.11328397

>>11328222
You need the total volume.

(Percent/100)*Volume/Price

Assuming both the bottles you listed are the same volume, you get 7.7% more alcohol with the second one.

>> No.11328420

In case I'm asleep once this thread hits the bump limit, here's the final version (right now):

>what is /sqt/ for
Questions relating to math and science, plus appropriate advice requests.
>where do I go for other SFW questions and requests?
>>>/wsr/ , >>>/g/sqt , >>>/diy/sqt , >>>/adv/ , etc.
>pdfs?
libgen.is (Warn me if the link breaks.)
>book recs?
https://sites.google.com/site/scienceandmathguide/
https://4chan-science.fandom.com/wiki//sci/_Wiki
>how do I post math symbols?
https://i.imgur.com/vPAp2YD.png
>a google search didn't return anything, is there anything else I should try before asking the question here?
https://scholar.google.com/
>where do I look up if the question has already been asked here?
>>/sci/
https://boards.fireden.net/sci/
>how do I optimize an image losslessly?
https://trimage.org/
https://pnggauntlet.com/

Question asking tips and tricks:
>attach an image
>look up the Tex guide beforehand
>if you've made a mistake that doesn't actually affect the question, don't reply to yourself correcting it. Anons looking for people to help usually assume that questions with replies have already been answered, more so if it has two or three replies
>ask anonymously
>check the Latex with the Tex button on the posting box
>if someone replies to your question with a shitpost, ignore it

Resources:
Good charts: https://mega.nz/#F!40U0zAja!cmRxsIoiLFZ_Mvu2QCWaZg
Shitty charts: https://mega.nz/#F!NoEHnIyT!rE8nWyhqGGO7cSOdad6fRQ (Post any that I've missed.)
Verbitsky: https://mega.nz/#F!80cWBKxC!ml8ll_vD2Gbw4I1hSLylCw
Graphing: https://www.desmos.com/
Calc problems: https://www.wolframalpha.com/

>> No.11328422

>>11328420
Sleep good <3

>> No.11328451

>>11328397
thanks, I think I understand

>> No.11328660

Suppose the |f| is bounded by M. Then if we have g = f^2(x), we know
g(x)-g(y) = (f(x) + f(y)) (f(x) - f(y)) <= 2M(f(x) - f(y))
How do I get to the inequality?

>> No.11328681
File: 87 KB, 801x366, 1579807849861.png [View same] [iqdb] [saucenao] [google]
11328681

>>11325627
probably because you interpreted wrong the thing there. also just dont give a fuck and keep going m8. shit happens

>> No.11328686

>>11328660
assuming you're taking [math] f^2(x) [/math] as [math] (f(x))^2 [/math], then the inequality just follows from the difference of two squares and [math] |f(x)|\leq M [/math]

>> No.11328688

>>11314603
how is the flower petals one stupid?

>> No.11328690
File: 42 KB, 1077x461, yo yo fuck.png [View same] [iqdb] [saucenao] [google]
11328690

probably not gonna get answers but here are 3 questions regarding physics.

>> No.11328695

>>11328660
I don't think this inequality always holds. Clearly f(x) <= M, f(y) <= M, f(x) + f(y) <= 2M. But multiplying by f(x)-f(y) will only hold the inequality if f(x) > f(y). I don't know the context this is in so maybe this is already given.

>> No.11328783

>>11328391
fuck, i kept subconciously thinking something about that bit was off.. Is this errata, or does it have something to do with:
>>11328681
why is that like that anyway? you think it's an accident?

>also just dont give a fuck and keep going m8. shit happens
sound advice, usually when I resort to shitposting it means I'm just moving on and hoping some kind anon helps me out when I check the thread again later.

Really curious as to why the author changed the index there though, and how that may have impacted the rest of the problem. Does this have anything to do with the formula for [math] \sum n^2 [/math] being wrong?

>> No.11328904

>>11328783
i dont know what did he mean but you should (know) because you already got from line 1 to 2. he probably meant to write eighter k=1 or k=(-1)
I wont do all the calculation. What is this equation for anyways? subject?

>> No.11328947

>>11328783
i didi the calculation and the n^3 term is indeed wrong. correct should be 10n^3/3
the other mistake i found gisves you a solution by induction: this book is shitty and wrong multiple times see>>11328391 too

>> No.11328954

>>11328947
shit i forgot to divide it by 5. 2n^3/3

>> No.11328956

>>11328690
"palied" meants to say applied

>> No.11328963
File: 10 KB, 431x107, help.png [View same] [iqdb] [saucenao] [google]
11328963

This equality is not obvious to me! I tried manipulating the left side algebraically into the right side but I couldn't figure it out. Please help?

>> No.11328964

>>11328947
>this book is shitty and wrong multiple times
fuck that sucks, lol. I got from line 1 to 2 with my index remaining at k, not k-1. After I noticed a mistmatch between my scratch work answer and the author's, I went back and copied his second to last line without realizing his mistakes. next time I'll be more careful / critical when doing so

should I post this on stack exchange so future retards don't get BTFO'd by this mistake as well?

>> No.11328981

>>11328964
>stack exchange
sure. or just write it in the copy if you got it form the library or write to the editor of the book

>> No.11328996
File: 16 KB, 640x589, dubass.png [View same] [iqdb] [saucenao] [google]
11328996

>>11328963
wow. imagine being so dumb and underage.

>> No.11329006

>>11328981
i bought it (it was only $20 and I already stare at a screen enough, plus I find unplugging and just relaxing with a hardcover comfy). it got good reviews everywhere, but memes be memes. this is my first encounter in this book with undocumented errata, to be fair

i did buy a laser print though, so from now on I'm just gonna print and self bind. i might do that and just switch to the fourth edition, which has errata updated on the website from the author at least.

I'll write it up to stack exchange later so the cyber world will know

>> No.11329150
File: 9 KB, 231x250, plebb.jpg [View same] [iqdb] [saucenao] [google]
11329150

How do I write proportionality?
""If x increases, y decreases"" in equation form.

>> No.11329231

>>11328996
I was trying to derive the quotient rule ;(

>> No.11329243

>>11329150
[math] f(x,y) = \frac{y}{x} [/math]

>> No.11329673

>>11329150
If it's an inverse relationship then you can say [math]x \propto \frac{1}{y}[/math]

>> No.11330108

>>11329150
> How do I write proportionality?
y∝x
> If x increases, y decreases
dy/dx<0
"Proportional" means that y=kx for some constant k (not necessarily positive), i.e. the graph is a straight line through the origin. "Inversely proportional" means that y=k/x for some constant k, i.e. the graph of y against 1/x is a straight line through the origin.

>> No.11330337

>>11329150
x*y=k
k is a constant